Let $x_1, \ldots , x_{100}$ be nonnegative real numbers such that $x_i + x_{i+1} + x_{i+2} \leq 1$ for all $i = 1, \ldots , 100$ (we put $x_{101 } = x_1, x_{102} = x_2).$ Find the maximal possible value of the sum $S = \sum^{100}_{i=1} x_i x_{i+2}.$ Proposed by Sergei Berlov, Ilya Bogdanov, Russia
Problem
Source: IMO Shortlist 2010, Algebra 3
Tags: inequalities, IMO Shortlist, n-variable inequality
18.07.2011 21:38
30.10.2011 20:56
$x_{i}+x_{i+1}+x_{i+2}\leq 1 $ and $ 0 \leq x_{i+2} \Rightarrow x_{i}x_{i+2}\leq x_{i+2}-x_{i+2}^2-x_{i+1}x_{i+2}$ $2S\leq \sum^{100}_{i=1}(2x_{i}-(2x_{i}^2+2x_{i}x_{i+1}))=\sum^{100}_{i=1}(x_{i}+x_{i+1})-(x_{i}+x_{i+1})^2=\sum^{100}_{i=1}(x_{i}+x_{i+1})(1-(x_{i}+x_{i+1}))\leq 100\cdot\frac{1}{4}=25 $ it follows that $S\leq \frac{25}{2}$ we can make equality setting $x_{2i}=\frac{1}{2}$ and $x_{2i-1}=0$ So the answer is $S= \frac{25}{2}$
03.06.2015 23:18
nice problem!! solution = by $x_i + x_{i+1} + x_{i+2} \leq 1$ and than AM-GM we have $x_i.x_{i+2}+x_{i+1}x_{i+3}\leq (1-x_{1+1}-x_{i+2})(x_{i+2})+(1-x_{1+1}-x_{i+2})(x_{i+2})= (1-x_{1+1}-x_{i+2})(x_{i+1}+x_{1+2})\leq \frac{1}{4}$ and so max. $S = \sum^{100}_{i=1} x_i x_{i+2}=\frac{25}{2}$ so we are done
16.03.2017 01:24
Yay solved an A3
12.06.2017 11:52
Cute little problem! IMO ShortList 2010 A3 wrote: Let $x_1, \ldots , x_{100}$ be nonnegative real numbers such that $x_i + x_{i+1} + x_{i+2} \leq 1$ for all $i = 1, \ldots , 100$ (we put $x_{101 } = x_1, x_{102} = x_2).$ Find the maximal possible value of the sum $S = \sum^{100}_{i=1} x_i x_{i+2}.$ Proposed by Sergei Berlov, Ilya Bogdanov, Russia Answer: the desired maximal value is $\boxed{\frac{25}{2}}$. For $x_{2i-1}=0$ and $x_{2i}=\frac{1}{2}$ we see that equality is achieved; next we show that this is an upper bound. Claim: Let $a, b, c, d$ be non-negative real numbers with $a+b+c \leqslant 1$ and $b+c+d \leqslant 1$. Then, $$ac+bd \leqslant \frac{1}{4}.$$ (Proof) Notice that $$ac+bd \leqslant ac+d(1-d-c)=d(1-d)+c(a-d),$$and similarly, $$ac+bd \leqslant a(1-a-b)+bd=a(1-a)-b(a-d).$$Note that for $x \in [0,1]$ we have $x(1-x) \le \frac{1}{4}$ and one of the inequalities $a \geqslant d$ or $d \geqslant a$ must hold; hence the claim is valid. $\blacksquare$ Evidently, from the previous claim we see that $$\sum^{100}_{i=1} x_ix_{i+2}=\frac{1}{2} \sum^{100}_{i=1} (x_ix_{i+2}+x_{i+1}x_{i+3}) \leqslant \frac{100}{8}=\frac{25}{2},$$as desired. $\blacksquare$
17.06.2018 20:44
10.10.2019 23:11
We claim the maximal value is $\boxed{25/2}$, achieved when $x_i=1/2$ when $i$ even and $0$ when $i$ odd. We'll now show that $S\le 25/2$. Claim: We have \[x_1x_3+x_2x_4\le 1/4.\] Proof: Let $a=x_1+x_2+x_3$ and $b=x_2+x_3+x_4$. Then, \begin{align*} x_1x_3+x_2x_4 &= x_1(a-x_1-x_2)+x_2(b-x_2-a+x_2+x_3) \\ &= a(x_1-x_2)+bx_2-x_1^2. \end{align*}If $x_1\ge x_2$, then we can say \[x_1x_3+x_2x_4\le 1\cdot(x_1-x_2)+1\cdot x_2-x_1^2=x_1-x_1^2\le 1/4.\]If $x_1\le x_2$, then since $a\ge x_1+x_2$, we have \[x_1x_3+x_2x_4\ge(x_1+x_2)(x_1-x_2)+1\cdot x_2-x_1^2=x_2-x_2^2\le 1/4,\]so in all cases, $x_1x_3+x_2x_4\le 1/4$, as desired. $\blacksquare$ The claim clearly generalizes to $x_ix_{i+2}+x_{i+1}x_{i+3}\le 1/4$, so summing over odd $i$ gives the desired result.
11.01.2020 16:35
Cool! orl wrote: Let $x_1, \ldots , x_{100}$ be nonnegative real numbers such that $x_i + x_{i+1} + x_{i+2} \leq 1$ for all $i = 1, \ldots , 100$ (we put $x_{101 } = x_1, x_{102} = x_2).$ Find the maximal possible value of the sum $S = \sum^{100}_{i=1} x_i x_{i+2}.$ Proposed by Sergei Berlov, Ilya Bogdanov, Russia We have $$x_i \cdot x_{i+2} + x_{i+1}\cdot x_{i+3} \le (1-x_{i+1} - x_{i+2}) \cdot (x_{i+2}) + (1 - x_{i+1}-x_{i+2})\cdot x_{i+1} = (1-x_{i+1}-x_{i+2})\cdot (x_{i+1}+x_{i+2}) \le \frac{1}{4} \iff S \le 50 \cdot \frac{1}{4} = \frac{25}{2} \blacksquare$$
10.02.2020 11:21
08.06.2020 04:53
redacted.
23.08.2020 02:39
Quite surprisingly, the dumbest bound I can think of works. Note that $x_1x_3+x_2x_4\leq (1-x_1-x_2)x_3+(1-x_1-x_2)x_2=(1-x_1-x_2)x_2x_3\leq \frac{1}{4}.$ Since there are $50$ pairs, the upper bound is $\frac{25}{2}.$ To show that this works, take the construction $(\frac{1}{2},0,\frac{1}{2},0,\ldots).$
26.06.2021 22:51
let $x_i+x_{i+1}=z_i$, then $\forall i$ note the inequality \[x_{i-1}x_{i+1}+x_ix_{i+2} \leq (1-x_i-x_{i+1}) x_{i+1} + x_i (1-x_i x_{i+1} = (x_i+x_{i+1})-(x_i+x_{i+1})^2= z_i\cdot (1-z_i) \leq \left(\frac{z_i+(1-z_i)}{2}\right)^2=\frac{1}{4}\]Tthis clearly means that \[S\leq 50\cdot \frac{1}{4} = \frac{25}{2}\]equality is achievable when $x_{odd}=0$ and $x_{even}=\frac{1}{2}$. $\textbf{Remark }$ : I was quite surprised that there is no solution where you separately consider odds and evens, instead the only solutions seems to be brutally mashing together adjacent terms. Given $E=$ sum of the evens, it is very difficult to bound the $\sum x_{2i}x_{2i+2}$, it is not true that setting all equal is optimal , nor is setting as many $\frac{1}{2}$s. The setting as many $\frac{1}{2}$s as possible seems optimal for large enough $E$, but for small E it fails (for example $\frac{1}{3},\frac23,\frac13$ vs. $\frac{1}{2},\frac{1}{2},\frac13$.), so it seems that the philosophically more motivated solution of seperately considering odds and evens fails
21.01.2022 17:03
Lemma: $a,b,c,d$ nonnegativa real numbers.If $a+b+c \leq 1$ and $b+c+d \leq 1$ $ \implies$ $ ac + bd \leq \frac{1}{4}$ Proof:$$ac \leq c(1-b-c)$$$$ bd \leq b(1-b-c)$$$$ac+bd \leq (b+c)(1-b-c) \leq \frac{1}{4}.$$By lemma $S = \sum^{100}_{i=1} x_i x_{i+2} \leq \frac{50}{4}.$
23.01.2022 22:44
Answer: $\frac{50}{4}$, and the example is when $x_{2k}=1/2$ otherwise $x_i=0$ Claim:For any $i=1,2,\dots,100$: $x_ix_{i+2}+x_{i+1}x_{i+3} \leq \frac{1}{4}$ Proof: $$x_ix_{i+2}+x_{i+1}x_{i+3} \leq x_{i+2}(1-x_{i+2}-x_{i+1})+x_{i+1}(1-x_{i+2}-x_{i+1})$$$$x_ix_{i+2}+x_{i+1}x_{i+3} \leq (1-x_{i+2}-x_{i+1})(x_{i+2}+x_{i+1})\leq \frac{1}{4}$$The last part is by AM-AG. $\square$ Finally: $S = \sum^{100}_{i=1} x_i x_{i+2} \implies 2S = \sum^{100}_{i=1} x_i x_{i+2}+x_{i+1} x_{i+3} \leq \frac{100}{4} \implies S\leq \frac{50}{4} $ So we are done.
15.04.2022 15:50
Note that we have $$x_ix_{i+2}+x_{i+1}x_{i+3}\leq x_{i+2}(1-x_{i+1}-x_{i+2})+x_{i+1}(1-x_{i+1}-x_{i+2})=(x_{i+1}+x_{i+2})(1-(x_{i+1}+x_{i+2})) \leq \frac{1}{4},$$so $S$ has maximum $\frac{25}{2}$, achieved by setting $x_{\text{odd}}=\frac{1}{2}$ and $x_{\text{even}}=0$. $\blacksquare$
06.08.2022 20:25
We claim that the maximum is $\frac{25}{2}$ when $(\frac{1}{2},0,\frac{1}{2},0,\dots).$ Since \[x_ix_{i+2}+x_{i+1}x_{i+3}\le (1-x_{i+1}-x_{i+2})x_{i+2}+x_{i+1}(1-x_{i+1}-x_{i+2})=(1-x_{i+1}-x_{i+2})(x_{i+1}+x_{i+2})\le \frac{1}{4}\]and there are $50$ of these pairs, $S\le \frac{25}{2}.$
01.01.2023 00:43
We claim that the answer is 12.5. This can be achieved by alternating 1/2 and 0. Consider four consecutive terms in the circle, $a,b,c,d$. Claim: $ac+bd\leq 1/4.$ We have $a+b+c\leq 1$ and $b+c+d\leq 1$. WLOG that $a\geq d$. Then, we have $b+c\leq 1-a$. Additionally, since $a\geq d$, and $b+c \leq 1-a$, $ac+bd$ is maximized when $c=1-a$ and $b=0$, so this becomes $a(1-a)$, and since $a(1-a)\leq 1/4$, this shows the claim. Summing this claim over every pair of terms in the sum gives that it is at most 12.5, so we are done.
24.08.2023 02:07
..what? It's evident by the given condition and AM-GM that $$x_kx_{k+2}+x_{k+1}x_{k+3}\le x_{k+2}(1-x_{k+1}-x_{k+2})+x_{k+1}(1-x_{k+1}-x_{k+2})=(x_{k+1}+x_{k+2})(1-(x_{k+1}+x_{k+2}))\le\frac14;$$in particular, over all 100 terms this occurs 50 times, so the maximum is $\frac{50}2,$ with equality at $x_{2n+1}=\frac12,x_{2n}=0$.
11.09.2023 17:06
We claim that the maximum possible value is $\frac{25}{2}$, which is achievable by alternating $\frac{1}{2}$ and $0$. Claim: Suppose that $x_1 + x_2 + x_3 \le 1, x_2 + x_3 + x_4 \le 1$. Then $x_1x_3 + x_2x_4 \le \frac14$. Proof. WLOG we can assume that $x_1 + x_2 + x_3 = x_2 + x_3 + x_4 = 1$, so $x_1 = x_4$. As such, this simplifies as $x_1(x_2 + x_3) = x_1(1 - x_1) \le \frac{1}{4}$. $\blacksquare$ Applying this $50$ times on $x_{2k+1}x_{2k+3} + x_{2k+2}x_{2k+4}$ finishes.
18.11.2023 14:41
Can anyone identify what is wrong with my Sol.
21.03.2024 21:30
i will grind and grind and grind. allan needs to stop being arrogant. allan needs to get better at math. after some thinking i came up with this: Consider the four-variable case with $(a,b,c,d)$. Using only $a+b+c\le 1$ and $b+c+d\le 1$, we can obtain $b+c\in [0,1]$ and \[ac+bd\le (b+c)(1-(b+c))\le \frac{1}{4}.\]So the answer for the four variable case is \[ac+bd+ca+db\le \frac{1}{2}.\]Extending the argument, \[x_ix_{i+2}+x_{i+1}x_{i+3}\le (x_{i+1}+x_{i+2})(1-x_{i+1}-x_{i+2})\le \frac{1}{4}\]hence summing yields $2S\le 25$ or $S\le \frac{25}{2}$. This is achieveable when $x_{2k}=\frac{1}{2}$ and $x_{2k+1}=0$. $\blacksquare$
01.05.2024 23:12
cute
02.12.2024 06:50
Easier than A2 For $1\leq k\leq 50$, let $t=\max(x_{2k-1},x_{2k+2})$ and note that $t+x_{2k+1}+x_{2k}\leq 1$. So we have, $$x_{2k-1}x_{2k+1}+x_{2k}x_{2k+2} \leq t(x_{2k+1}+x_{2k})\leq t(1-t)=\frac 14-\left (t-\frac 12\right)^2\leq \frac 14.$$And, $$S=\sum_{k=1}^{50} (x_{2k-1}x_{2k+1}+x_{2k}x_{2k+2})\leq \sum_{k=1}^{50} \frac 14=\frac{25}2.$$Equality holds, for instance, when $x_i=\begin{cases} 1/2 & i \text{ odd} \\ 0 & i \text{ even} \end{cases}$. This proves that the maximal value is $\frac{25}2$.
31.12.2024 20:00
The answer is $\frac{25}2$. Equality is achieved when $a_{2k-1} = \frac 12$ and $a_{2k} = 0$ for all $1 \leq k \leq 50$. To show the bound, write \begin{align*} \sum_{i=1}^{100} x_ix_{i+2} &= \sum_{k=1}^{50} x_{2k-1}x_{2k+1} + x_{2k}x_{2k+2} \\ &\leq \sum_{k=1}^{50} x_{2k+1}\left(1-x_{2k}-x_{2k+1}\right) + x_{2k}\left(1-x_{2k+1}-x_{2k}\right) \\ &\leq \sum_{k=1}^{50} \left(x_{2k}+x_{2k+1}\right)\left(1-x_{2k}-x_{2k+1}\right) \\ &\leq 50 \cdot \frac 14 = \frac{25}2. \end{align*}Remark: The equality case almost fully motivates the solution because we want a bound that is sharp when $a_{2k} + a_{2k+1} = \frac 12$ is constant.